Question

Marginal Cost = ∂TC/ ∂Q = (∂ 5Q2 + 10Q +180) / ∂Q How do I...

Marginal Cost = ∂TC/ ∂Q = (∂ 5Q2 + 10Q +180) / ∂Q How do I get to the = 10Q +10 solution? Please show the steps.

0 0
Add a comment Improve this question Transcribed image text
Answer #1

Cot efu el Cost), we take S O Hence ,

Add a comment
Know the answer?
Add Answer to:
Marginal Cost = ∂TC/ ∂Q = (∂ 5Q2 + 10Q +180) / ∂Q How do I...
Your Answer:

Post as a guest

Your Name:

What's your source?

Earn Coins

Coins can be redeemed for fabulous gifts.

Not the answer you're looking for? Ask your own homework help question. Our experts will answer your question WITHIN MINUTES for Free.
Similar Homework Help Questions
  • Consider a firm that has cost function of: TC = 24 + 10Q + 1.5 Q^2...

    Consider a firm that has cost function of: TC = 24 + 10Q + 1.5 Q^2 a) Draw the average cost and marginal cost curves for this firm (label with equation). b) Draw the supply curve of this firm c) What are the profits of the firm if the price of output is $44?

  • TC = 500 + 10Q + 0.05 Q*Q What is the firm's fixed cost What is...

    TC = 500 + 10Q + 0.05 Q*Q What is the firm's fixed cost What is the firm's average variable cost?

  • Consider a competitive firm with total costs given by TC(q) = 100 + 10q + q...

    Consider a competitive firm with total costs given by TC(q) = 100 + 10q + q 2 The firm faces a market price p = 50. (a) Write expressions for total revenue TR and marginal revenue MR as functions of output q. (b) Write expressions for average total cost ATC, average variable cost AVC, and marginal cost MC as functions of output q. (c) For what value of output is ATC minimized?

  • For the given cost function, find TFC and MC, TC = 10,000 +10Q 1. TFC =...

    For the given cost function, find TFC and MC, TC = 10,000 +10Q 1. TFC = $10,000 & MC = $10 2. TFC = $10 & MC = $(10,000/Q) + $10 3. TFC = $10,000 & MC = $10,000 4. TFC = $10 & MC = $10 Pls explain how to get TFC & MC thanks!

  • Suppose that a firm has a short run, total cost function given by: TC= 1089 +10q...

    Suppose that a firm has a short run, total cost function given by: TC= 1089 +10q +9q2. 1. Determine the profit-maximizing quantity of production when price is $244. _____________________________________ q= 13 2. Calculate the price at which this firm breaks even (i.e. profit = $0). _____________________________________ $208 3. Calculate the price at which this firm shuts down in the short run. _____________________________________ $10 The answers are given but can you show how to get them step by step.

  • Consider a competitive rm with total costs given by TC(q) = 100 + 10q + q^2,...

    Consider a competitive rm with total costs given by TC(q) = 100 + 10q + q^2, The firm faces a market price p = 50. (a) Write expressions for total revenue TR and marginal revenue MR as functions of output q. (b) Write expressions for average total cost ATC, average variable cost AVC, and marginal cost MC as functions of output q. (c) For what value of output is ATC minimized? (d) Find the profit maximizing level of output q...

  • The total cost (TC) of producing computer software diskettes (Q) is given as: TC = 100...

    The total cost (TC) of producing computer software diskettes (Q) is given as: TC = 100 + 10Q. What is the; a. fixed cost (FC), b. variable cost (VC), c. marginal cost (MC), d. average fixed cost (AFC), e. average variable cost (AVC), f. average total cost (ATC)?

  • c) TC() -40q+ 200 Marginal cost MC Average cost AC- Returns to scale Marginal cost: MC-...

    c) TC() -40q+ 200 Marginal cost MC Average cost AC- Returns to scale Marginal cost: MC- Average cost: Returns to scale: Name: ECP 4413-Government& Business Spring 2019 Problem Set 1 Due start of class on Tuesday, Jan 22 Please print this out and put your answers here. Showing your work may get you partial credit for incorrect answers. I will not grade work tumed in on anything else. If you must miss class and cannot drop off your answers, email...

  • Hi please write everything out legibly thank you Using the cost function TC 3 10Q+50, and...

    Hi please write everything out legibly thank you Using the cost function TC 3 10Q+50, and a production function Q KAL, K-85, L 120, determine the following 1. Quantity of output 2. Average product of labor 3. Average product of capital 4. Marginal product of labor 5 Marginal product of capital 6. Fixed cost and average fixed cost 7. Variable cost and Average variable cost 8. Total cost and average total cost 9 If capital rental rater 5 and wage...

  • For the given cost function, find TFC and MC, TC = 10,000 +10Q 1. TFC =...

    For the given cost function, find TFC and MC, TC = 10,000 +10Q 1. TFC = $10,000 & MC = $10 2. TFC = $10 & MC = $(10,000/Q) + $10 3. TFC = $10,000 & MC = $10,000 4. TFC = $10 & MC = $10

ADVERTISEMENT
Free Homework Help App
Download From Google Play
Scan Your Homework
to Get Instant Free Answers
Need Online Homework Help?
Ask a Question
Get Answers For Free
Most questions answered within 3 hours.
ADVERTISEMENT
ADVERTISEMENT
ADVERTISEMENT